Unit 4, Lesson 8: Tax and Tip newste
8.1: Notice and Wonder: The Price of Sunglasses
You are on vacation and want to buy a pair of sunglasses for $10 or less. You
find a pair with a price tag of $10. The cashier says the total cost will be
$10.45.
1. The tax is what percent of the original price?
2. An item that costs $20 would have a tax of how much?
3. Ray Bans cost $852.00, how much would the total be after tax?

Answers

Answer 1

1. The tax percentage will be 4.5%.

2. An item that cost $20 would have a tax of $9.

3. Ray Bans cost $852.00, the total cost after tax would be $1,223.

Define percentage.

A value or ratio stated as a fraction of 100 is known as a percentage in mathematics. Percent is derived from the Latin word per centum, which means "by the hundred." In the sixteenth century, the Latin expression made its way into English. Although the abbreviations "pct.", "pct.", and occasionally "pc" is also used, the percentage sign, "%," is frequently used to denote it. An amount that is a percentage is dimensionless and has no assigned measurement.

Solution Explained:

Given here,

The original cost of the Sunglasses = $10

Total cost = $10.45

Tax amount = 10.45 - 10 = $0.45

1. Tax % = [tex]\frac{0.45}{10} X 100[/tex] = 4.5%

2. Using the tax percentage from (1),

Tax for an item that costs $20 is

= [tex]\frac{45}{100} X 20[/tex] = $9

3. Ray Bans cost = $852

Tax = [tex]\frac{45}{100} X 852[/tex] = $371.25

Total after-tax = 852 + 371.25 = $1,223

To learn more about percentage, use the link given
https://brainly.com/question/19247356
#SPJ9


Related Questions

All stereos are 20
%
off of the original price. If stereos normally cost $160, what is the sale price?

Answers

20% of 160

160 divided by 10 = 16

16x2 = 32

160-32= $128

Answer:

128 dollars

Step-by-step explanation:

hope it helped! :)

In the diagram, the lines L and M appear to be parallel. How do you know for sure?​

Answers

Proved  that the line L and M are parallel lines

Here we have to prove that L and M are parallel

One angle at the point at F is given that 139 degrees

We know sum of angles on a straight line is 180 degrees

Then the other angle will be

139 + a = 180

a = 180 - 139

a = 41 degrees

Now consider the triangle and find the value of x

The sum of interior angles of a triangle is 180 degrees

then

(3x+8) + (3x) + (9x+7) = 180

3x+8 +3x +9x + 7 = 180

15x + 15 = 180

15x = 180 - 15

15x = 165

x = 165/15

x  = 11

Then the value of angle 3x +  8 = 3 × 11 + 8

= 33 + 8

= 41 degrees

Before that we have solved the angle at f = 41 degrees, therefore both are equal angles  which is called corresponding angles

Therefore, the line L and M are parallel

Hence, proved  that the line L and M are parallel lines

Learn more about parallel lines here

brainly.com/question/12274210

#SPJ9

Write the equation of the line that passes through the points (1,-3) and (3,3)

Answers

The equation of line is y = 3x - 6

Define Equation of line.

A line in geometry is a collection of points that can be stretched infinitely in two different ways. To put it another way, a line is created by continually extending the two end points in any direction. As a result, we can assert that a line lacks end points. However, because it just has length and no width, it is a one-dimensional geometric form. Based on the information provided, we can create multiple forms of equations for lines algebraically, such as point slope form, slope-intercept form, and two point form. Two Point Form Formula,

          y - y₁ = m (x - x₁)  

We have the given points,

(1, -3) and (3, 3)

Let, (x₁ , y₁) = (1, -3)     and,

(x₂, y₂) = (3, 3)

Formula for equation of line is,

y - y₁ = m (x - x₁)

Where m is slope and for that we have formula for slope is

m =  (y₂ - y₁) / (x₂ - x₁)

plug in the values,

m = (3 - (-3) ) / (3 - 1)

m = 6 / 2

m = 3

Now, we get the slope. Now, just put the values in equation of line formula

y - y₁ = m (x - x₁)

y - (-3) = 3 (x - 1)

y + 3 = 3x - 3

y = 3x - 6

Hence, the equation of line is y = 3x - 6

To read more about Equation of line.

https://brainly.com/question/19417700

#SPJ9

8–7. The block brake conit of a pin-connected lever and
friction block at B. The coefficient of tatic friction between
the wheel and the lever i m = 0. 3, and a torque of 5 N # m
i applied to the wheel. Determine if the brake can hold the
wheel tationary when the force applied to the lever i
(a) P = 30 N, (b) P = 70 N

Answers

If the brake can hold the wheel stationary when the force applied to the lever i

(a) P = 30 N     -> No

(b) P = 70 N     -> Yes

To hold lever,

x + Σ[tex]M_{O} = 0[/tex]             [tex]F_{B}(0.15) - 5 = 0[/tex]            [tex]F_{B} = 33.333 N[/tex]

Require

[tex]N_{B} = \frac{33.333}{0.3}[/tex]

      = 111.1 N

Lever,

x + Σ[tex]M_{A} = 0[/tex]            [tex]P_{Reqd.}[/tex](0.6) - 111.1(0.2) - 33.333(0.5) = 0

[tex]P_{Reqd.}[/tex] = 39.8 N

(a) P = 30 N < 39.8 N           No

(b) P = 70 N > 39.8 N           Yes  

To learn more about force here:

https://brainly.com/question/7362815

#SPJ4

1. Determine the solution for the system of equations shown in this graph. Enter your answer as a coordinate pair without any spaces, infinite solutions, or no solutions.

Answers

The solution for the system of equations shown in this graph is (-2, 1)

In this question, we need to determine the solution for the system of equations shown in this graph.

From given graph of system of equations we can observe that the  system of equations represent lines intersected at point.

This means that the system of equations has a solution.

The point of intersection of given line is (-2, 1)

Therefore, the solution for the system of equations shown in this graph is (-2, 1)

Learn more about the  system of equations here:

https://brainly.com/question/24065247

#SPJ1

Given the explicit formula u(n) = 4n - 1, where u(1) = 1, find the recursive formula for u(n).
A. u(1) = 1 and u(n + 1) = 4u(n), for n = 1, 2, 3, . . .
B. u(1) = 1 and u(n + 1) = u(n) + 4, for n = 1, 2, 3, . . .
C. u(1) = 1 and u(n + 1) = u(n) + 8, for n = 1, 2, 3, . . .
D. u(1) = 1 and u(n + 1) = 8u(n), for n = 1, 2, 3, . . .

Answers

The Recursive formula for the given explicit formula is u(1) = 1 and u(n+1) = u(n) + 4 , for n = 1 , 2 , 3 ....  , the correct option is (b) .

In the question ,

it is given that ,

the explicit formula is u(n) = 4n - 1 .

and u(n) = 1 ,

we need to find the recursive formula ,

We substitute n + 1 in place of n , in the equation u(n) = 4n - 1

after substitution , we get

u(n+1) = 4(n + 1) - 1

u(n+1) = 4n + 4 - 1

u(n+1) = 4n + 3

Subtracting u(n) = 4n - 1 from u(n+1) = 4n + 3  ,

we get

u(n+1) - u(n) = 4n + 3 - (4n - 1)

u(n+1) - u(n) = 4n + 3 - 4n + 1

u(n+1) - u(n) = 3 + 1

u(n+1) - u(n) = 4

u(n+1) = u(n) + 4

Therefore , The Recursive formula for the given explicit formula is u(1) = 1 and u(n+1) = u(n) + 4 , for n = 1 , 2 , 3 ....

Learn more about Recursive Formula here

https://brainly.com/question/1827481

#SPJ1

Solve for x. Help me and I will do anything for you.

Answers

x = 8

Step-by-step -e-x-p-l-a-n-a-t-i-o-n- below

The angles shown are vertical angles

Vertical angles are congruent (equal to each other)

This means that 8x + 36 = 5x + 60

Using this equation we can solve for x

8x + 36 = 5x + 60  

==> subtract 36 from both sides

8x = 5x + 24

==> subtract 5x from both sides

3x = 24

==> divide both sides by 3

x = 8

Answer:

[tex]\huge\boxed{\sf x = 8}[/tex]

Step-by-step explanation:

From the diagram,

8x + 36 = 5x + 60 (Vertically opposite angles are equal)

Subtract 5x to both sides

8x - 5x + 36 = 60

3x + 36 = 60

Subtract 36 to both sides

3x = 60 - 36

3x = 24

Divide 3 to both sides

x = 24/3

x = 8

[tex]\rule[225]{225}{2}[/tex]

Simplify without a calculator 2x X 3x
- 2x²

Answers

Answer:

4x²

Step-by-step explanation:

2x × 3x - 2x²

= 6x² - 2x²

= 4x²

Find the simple interest of Rs. 20,000 for 5 years at 10% per annum? Also, at what time will Rs. 50,000 earn the same amount of interest at 5% of simple interest per annum?​

Answers

Answer:

below

Step-by-step explanation:

Simple interest =  20 000 * .1 * 5               (.1   is 10% in decimal)

at 1/2 the 10% interest it will take twice as long = 10 years

Probability: Discrete, Binomial, Normal
IB Math Applications SL
1. Jae Hee plays a game involving a
biased six-sided die. The score for
the game, X, is the number which
lands face up after the die is rolled.
The table shows the probability
distribution for X.
a. Find the exact value of p.
Score x
P(X=x)
Name: Somatar
Date:
-3
1
18
-1
P
Period:
0
3
18
1 point: Correct p-value
b. Jae Hee plays the game once. Calculate the expected score.
1
1
18
1 point: Correct substitution into formula
2 points: Correct substitution and answer
c. Jae Hee plays the game twice and adds the two scores together.
Find the probability Jae Hee has a total score of -3.
2
18

Answers

The precise value of p is 4/18. Jae Hee's expected score in the game is 1.833. Jae Hee has a 1/162 chance of receiving a total score of -3. The likelihood that something will happen is defined as probability.

How to calculate Probability ?Probability is calculated by dividing the number of possible outcomes by the total number of outcomes. Probability and odds are distinct concepts. Odds are the probability that something happens divided by the probability that it does not happen. Probability is a branch of mathematics that deals with numerical representations of the likelihood of an event occurring or a statement being true.

Therefore,

Value of p is,

∈p = 1

1/18 + p + 3/18 + 1/18 + 2/18 + 7/18 = 1

14/18 + p = 1

p = 4 / 18

So the exact value of p is 4/18

Expected score of the game :

∈(x) = ∈xi pi

= -3 (1/18) - 1 (4/18) + 0 (3/18) + 1 (1/18) + 2 (2/18) + 5 (7/18)

= 33/18

= 1.833

Expected score of the game Jae Hee plays is 1.833

Calculate Probability of Jae Hee getting total score of -3

p (total score -3 )

= p(x= -3) and p(x=0) + p(x=0) and p (x=3)

= 1/18 × 1/18 + 1/18 × 1/18

=1/162

Probability of Jae Hee getting total score of -3 is 1/162

To learn more about Probability, refer to:

brainly.com/question/24756209

#SPJ13

Answer:

Step-by-step explanation:

find the number that is exactly between 1/10 and 2/5

Answers

Answer:

1/4

Step-by-step explanation:

[tex] \frac{1}{2} ( \frac{1}{10} + \frac{2}{5} )[/tex]

[tex] \frac{1}{2} ( \frac{1}{10} + \frac{4}{10} ) = \frac{1}{2} ( \frac{5}{10} ) = \frac{1}{2} ( \frac{1}{2} ) = \frac{1}{4} [/tex]

What is the degree of the polynomial below?
x +32x² + 4x² - 4x
A.
7
B.
6
C.
5
D.
4

Answers

Answer:

2

Step-by-step explanation:

Degree is the largest exponent        = 2   (as written)

In a class of 29 students, 9 play an instrument and 14 play a sport. There are 8 students who do not play an instrument or a sport. What is the probability that a student does not play an instrument given that they play a sport?.

Answers

The probability of a student playing a sport but not playing an instrument is around 0.4286.

The likelihood of a student not playing an instrument, provided they play a sport, is:
[tex]Probability=\dfrac{Number\ of\ students\ who\ play\ a\ sport\ but\ do\ not\ play\ an\ instrument​}{Number of \students\ who\ play\ a\ sport}[/tex]

It is known that 14 students participate in a sport, while 8 students do not engage in either a musical instrument or a sport. Thus, the number of students who play a sport but do not play an instrument is:

Number of students who play a sport but do not play an instrument = 14- 6
                                                                                                                 = 6
Now, let's calculate the probability:

Probability = 6/14
                  = 0.4286

Rounded to four decimal places, the likelihood of a student playing a sport but not playing an instrument is approximately 0.4286.

Learn more about  probability here:

https://brainly.com/question/15844335

#SPJ12

Which system of linear inequalities is graphed?

A.
x<-3
y ≤-x+1

B.
x<-3
y ≤-x-1

C.
x<-2
y ≤-x-1

D.
x ≤-3
y ≤-x-3

Answers

Answer:

A x<-3

y ≤-x+1

Step-by-step explanation:

x<-3

y ≤-x+1

the breaking strength x of a certain rivet used in a machine engine has a mean 5000 psi and standard deviation 400 psi. a random sample of 36 rivets is taken. consider the distribution of , the sample mean breaking strength. what is the probability that the sample mean falls between 4800 psi and 5200 psi?

Answers

The probability that the sample mean falls between 4800 psi and 5200 psi is 0. 0.9973.

Probability refers to the numerical descriptions of how likely an event is to occur. For the given information, the mean µ = 5000 psi and standard deviation σ = 400 psi.

For a random sample, n = 36, in order to determine the probability that the sample mean falls between 4800 psi and 5200 psi, the z test statistics for the range must be calculated.

z = (x - µ)/ (σ/√n) = (4800 - 5000)/ (400/√36) = -3

z = (x - µ)/ (σ/√n) = (5200 - 5000)/ (400/√36) = 3

Hence, the probability that the sample mean falls between 4800 psi and 5200 psi would be p ( -3< z < 3).

From the table,

p ( -2.4 < z < 2.4) = 0.9973

Learn more about Probability:

https://brainly.com/question/24756209

#SPJ4

a baseball player averages 2 hits per game. The player already has 18 hits this season. another player has 22 hits so far this season and averages 1 hit per game. After how many games will both players have the same number of hits?

Answers

Answer: 20

Step-by-step explanation:

i got 100

Last year at a certain high school, there were 125 boys on the honor roll and 50 girls on the honor roll. This year, the number of boys on the honor roll increased by 12% and the number of girls on the honor roll increased by 8%. By what percentage did the total number of students on the honor roll increase? round your answer to the nearest tenth (if necessary).

Answers

The total student increases by 10.85% from last year.

What is percentage?

A relative value indicating hundredth parts of any quantity is called percentage.

Given that, last there were 125 boys and 50 girls, and it increased by 12% and 8% next year,

Total students last year = 125+50 = 175

Boys increased = 125x112/100 = 140

Girls increased = 50x108/100 = 54

Total students current year = 140+54 = 194

Percentage increase = (194-175)/175x100 = 10.85%

Hence, The total student increases by 10.85% from last year.

For more references on percentage, click;

https://brainly.com/question/29306119

#SPJ1

Write a rate that represents the situation.


Need help on 5 & 6

Answers

Answer:

5.  (6 hours)/(1 day)

6.  (8 quarts)/(1 container)

Step-by-step explanation:

5.  The number line shows both hours and days.  It does not indicate why the relationship exists.  Perhaps the hours are the ones worked in a day.  What’s important is whether the relationship between the two is the same for all data pairs.  E.g., 12/2 = 6; 36/6 = 6; 60/6 - 5, and so forth.  They are all the same ratio, or rate: (6 hours)/(1 day)

6.  The number line shows both quarts and containers.  The  relationship between the two is the same for all data pairs.  E.g., 8/5,  16/10 or 8/5; 24/15 or 8/5; 32/20 or 8/5, and 40/25 or 8/5.   They are all the same ratio, or rate: (8 quarts)/(1 container).

When can you use proportional reasoning to solve a problem?
A. When two quantities add to 1
B. When one quantity is a constant multiple of the other
C. When the two quantities multiplied equal a constant
D. When one quantity is 1 more than the other

Answers

We can use proportional reasoning to describe that when the two quantities multiplied equal a constant.

Hence, Option (C) is correct.

What is a proportion?

A proportion is a type of ratio in which the denominator also contains the numerator. For example, the proportion of male deaths would be deaths to males divided by deaths to males plus deaths to females (i.e. the total population).

Let 'x' be the numerator and 'y' be denominator,

then one ratio is 'x/y' and its reciprocal by using the proportion would be 'y/x'

and then their multiplication will be

                               [tex]=\frac{x}{y}*\frac{y}{x} \\\\\=1[/tex]

Multiplication gave us '1', which is a constant.

Hence option (C) is the correct answer.

We can use proportional reasoning to describe that when the two quantities multiplied equal a constant.

To learn more about the proportions visit:

https://brainly.com/question/870035

#SPJ9

51.765 rounded 2 decimal places

Answers

The answer would be:
51.77

Answer:

The answer would be 52 if you rounded by the nearest whole number.

Step-by-step explanation:

Find your place

Look next door (to the minor digit)

Four or less just ignore

Five or more let it sore.

a farmer raises chickens and sheep on his farm. the ratio of the total number of legs of the chickens to the total number of legs of the sheep is 4:7. find the minimum number of chickens and sheep he has.

Answers

Answer:

8 chickens, and 28 sheep.

Step-by-step explanation:

The chickens have the ratio of 4. Chickens have 2 legs, so 4 x 2 = 8 chickens.

The sheep have the ratio of 7. Sheep have 4 legs, so 7 x 4 = 28 sheep.

May I have Brainliest please? My next rank will be the highest one: A GENIUS! Please help me on this journey to become top of the ranks! I only need 15 more brainliest to become a genius! I would really appreciate it, and it would make my day! Thank you so much, and have a wonderful rest of your day!

Sari made pancake batter. She has 1 cup of batter left. How much batter did she use?
If this problem cannot be solved with the information provided, write additional information
and then solve the problem.

Answers

Answer:

3 cups

Step-by-step explanation:

let's assume that sari made pancake batter in a jar.

1 jar is equivalent to 4 cups

so if 1 cup of batter is left, it can be concluded that Sari uses 3 cups of batter in making pancakes.

help please (see picture)

Also if it’s not clear cus of the smudge, it says “6” and “U”

Answers

The perimeter of the triangle is 64 when the triangle is ZTW.

Given that,

In the picture we have a triangle ZWT.

The U and V are mid points of WT and Y and X are midpoints of ZW.

We have to find the perimeter of the triangle.

We get the sides

ZT=22

We have,

TU/ZY= UV/YX= VW/XW  (Thales theorem)

6/8= a/12= 3/b

3/4=a/12= 3/b

Take,

3/4=a/12

a=3/4×12

a=3×3

a=9

Take,

3/4=3/b

b=3×4/3

b=4

The side TW=6+9+3=18

The side ZW=8+12+4=24

The perimeter of the triangle is sum of the sides.

The perimeter of the triangle=ZT+ZW+TW

The perimeter of the triangle= 22+24+18

The perimeter of the triangle= 64

Therefore, The perimeter of the triangle is 64 when the triangle is ZTW.

To learn more about perimeter visit: https://brainly.com/question/6465134

#SPJ1

help meeeeeeeeeeee pleaseee rnnnnn!!!!!!!!!!!!!!!!!!!!!!!!!!!!!!!!!!!!!!!!!!!!!!!!!!!!!!!!!!!!!!!!!!!!!!!!!!!!!help meeeeeeeeeeee pleaseee rnnnnn!!!!!!!!!!!!!!!!!!!!!!!!!!!!!!!!!!!!!!!!!!!!!!!!!!!!!!!!!!!!!!!!!!!!!!!!!!!!!

Answers

The completed table of coordinates for the exponential function can be  presented as follows;

x; -2,       -1,  0, 1,       2

y ; 16/9, 4/3, 1, 3/4, 9/16

Please find attached the graph of the function created with MS Excel

The correct graph is therefore, option D

What is an exponential function?

An exponential function is one that has the argumant as an exponent

The values in the table can be found as follows;

[tex]f(x) = \left(\frac{3}{4} \right)^x[/tex]

[tex]f(x) = \left(\frac{3}{4} \right)^{-2} = \dfrac{16}{9}[/tex]

[tex]f(x) = \left(\frac{3}{4} \right)^{-1} = \dfrac{4}{3}[/tex]

[tex]f(x) = \left(\frac{3}{4} \right)^{0} = 1[/tex] The y-intercept

[tex]f(x) = \left(\frac{3}{4} \right)^{1} = \dfrac{3}{4}[/tex]

[tex]f(x) = \left(\frac{3}{4} \right)^{2} = \dfrac{9}{16}[/tex]

The completed table is therefore;

x; -2,       -1,  0, 1,       2

y ; 16/9, 4/3, 1, 3/4, 9/16

The graph of the function ,  [tex]f(x) = \left(\frac{3}{4} \right)^x[/tex], therefore, has a y-intercept of 1, and a maximum value of 16/9 at x  = -2

The best cprect option  for the graph is therefore option  D

Learn more about exponential functions here:

https://brainly.com/question/11464095

#SPJ1


Sally adds 3.13 moles of argon to a 5.29 liter balloon that already contained 2.51 moles of argon. What
is the volume of the balloon after the addition of the extra gas?

Answers

The volume of the balloon after the addition of the extra gas is 11.87 liters

The initial volume of the balloon = 5.29 liters

Initial quantity of argon in the balloon = 2.51 mole

Here we have to use the unitary method

The volume of the 1 mole of argon = The initial weight  of the balloon / Initial quantity of argon in the balloon

Substitute the values in the equation

The volume of the 1 mole of argon = 5.29 / 2.51

= 2.11 liters

Additional argon added = 2.51 mole

Total quantity of argon = 2.51 + 3.13

= 5.64 mole

The volume of the 5.64 mole of argon = 5.64 × 2.11

= 11.87 liters

Hence, the volume of the balloon after the addition of the extra gas is 11.87 liters

Learn more about unitary method here

brainly.com/question/23423168

#SPJ1

The function c(x) = 14x + 13 represents the cost (in dollars) of renting a surfboard, where x is the number of hours you rent
the surfboard.
a. Graph the function and identify its domain and range.

Answers

The domain of this function c(x) = 14x + 13 using interval notation is (0, ∞).

The range of this function c(x) = 14x + 13 using interval notation is (13, ∞).

How to identify the domain and range any graph?

In Mathematics, the horizontal portion of any graph is used to represent all domain values and they are both read and written from smaller to larger numerical values, which simply means from the left of any graph to the right.

Additionally, the vertical extent of any graph represents all its range values and they are always read and written from smaller to larger numerical values, and from the bottom of a graph to the top.

By critically observing the graph of this function shown in the image attached below, we can reasonably infer and logically deduce the following:

Domain = (0, ∞).

Range = (13, ∞).

Read more on domain here: brainly.com/question/17196520

#SPJ1

help meeeeeeeeeeee pleaseee rnnnnn!!!!!!!!!!!!!!!!!!!!!!!!!!!!!!!!!!!!!!!!!!!!!!!!!!!!!!!!!!!!!!!!!!!!!!!!!!!!!

Answers

The amount owed at the end of the 6 years duration is $10865

How to determine the amount owed in six years?

The given parameters about the compound interest are

Principal Amount, P = $5300

Interest Rate, R = 8%

Time, t = 6 years

To calculate the amount owed, we make use of the following formula

A = P + CI

Where

Compound interest, CI = P(1 + R)^t - P

So, the equation becomes

A = P + P(1 + R)^t - P

Evaluate the like terms

A = P(1 + R)^t

In this question, the formula is given as

A = P(1 + r/n)^nt

Where

n = 12, i.e compounded monthly

Substitute the known values in the above equation

A = 5300 * (1 + 8%/12)^(6*12)

Express 8% as decimal

A = 5300 * (1 + 0.08/12)^(6*12)

Evaluate the sum

A = 5300 * (1.01)^(6*12)

Evaluate the exponent

A = 5300 * 2.05

Evaluate the product

A = 10865

Hence, the value of the amount after 6 years is $10865

Read more about compound interest at:

brainly.com/question/2455673

#SPJ1

I’ll give brainlist!!!
What is the slope?

Answers

The slope of the line is 1/3

First we have to choose two points from the graph

The first point = (1, -5)

The second point = (7, -3)

The slope of the line is the change is y coordinate with respect to the change in x coordinate of the function

The slope of the line m = [tex]\frac{y_2-y_1}{x_2-x_1}[/tex]

Where m is the slope of the line

[tex](x_1,y_1)[/tex] is the coordinate of the first point

[tex](x_2,y_2)[/tex] is the coordinate of the second point

Substitute the values in the equation

The slope of the line = (-3 - (-5) / (7 - 1)

= (-3+5) / 6

= 2 / 6

= 1/3

Hence, the slope of the line is 1/3

Learn more about slope of the line here

brainly.com/question/16180119

#SPJ1

I would like to know the answer..

Answers

Answer:

Ann’s work is correct

Step-by-step explanation:

PLEASE HELP!!!! I BEG YOU!!!
What is the solution to the system of equations graphed below? Give the answer as a coordinate point (x, y)

Answers

Answer:

(1,3)

Step-by-step explanation:

The solution to a system of equations graph is the point of intersection of the two lines.

In this case the point where both lines meet is at x = 1 and y = 3

hence the answer being (1,3)

Other Questions
if a couple that are both heterozygous carriers of the cystic fibrosis allele have children, the chance that both their first and their second child are carriers is 0.5 x 0.5, or 25%, as determined by the_______ For a lower tail test, the p-value is the probability of obtaining a value for the test statistic at least as. a researcher discussing the prevalence of eating disorders makes reference to what she says is a cohort effect for these disorders. what does this mean? find the missing terms of the arithmetic sequence __ -6 __ __ 15 __ 2. Mitosis is a continuous process not a series of separate events. What evidence did you observe to support this statement?. According to an architect's design, the new rectangle stables (housing for horses) will have total dimensions of 30 yards by 10 yards. The design includes a walkway between the stables where no horses can stay. The walkway will take up 45 yards ^ 2 Each horse requires a minimum of 12yards ^ 2. How many horses at most can live in the stables? which of the following is not found in the financing section of the cash budget? multiple choice question. borrowings cash deficiency repayments interest how many branches are there Write a letter to yourself in 10 years congratulating yourself on at least three (3) ways that you were heart healthy and the benefits of those choices. At least 7 sentences the powerful survival impulse that leads infants to seek closeness to their caregivers is called attachment. conservation. egocentrism. pruning. the project cost management knowledge area maps to which process group through the activities of estimating costs and budget determination? the mother of a 3-year-old asks a clinic nurse about appropriate and safe toys for the child. the nurse should tell the mother that the most appropriate toy for a 3-year-old is which? In rst, vt=60 in. what is the length of tx? enter your answer in the box. in. Table D. pH of Serial Dilutionswhat is the Predicted pH range of A2 (it doesnt give the liquid name)What is the Predicted pH range of A3 (no name of liquid given)Please HElp!!! qweschion: what are the factors for 60Explanation: HELP IM IN CLAS AS WE SPEEK! what attributes would a clastic sedimentary rock have if it had traveled far from its source and was carried in a current with uniform energy review your observations of the reactions of the metals with ions in solution including the hydrogen ion. which ion was the most reactive? a customer owns cumulative preferred stock (par value of $100) that pays an 8% dividend. the dividend has not been paid this year and was missed in the two previous years. if the company wants to pay a dividend to common shareholders, how much must the company pay this customer per share first? Use present tense or infinitive form for each sentence How do you find the mean absolute deviation step by step?.